What is Conditions: Definition and 1000 Discussions

In mathematical optimization, the Karush–Kuhn–Tucker (KKT) conditions, also known as the Kuhn–Tucker conditions, are first derivative tests (sometimes called first-order necessary conditions) for a solution in nonlinear programming to be optimal, provided that some regularity conditions are satisfied.
Allowing inequality constraints, the KKT approach to nonlinear programming generalizes the method of Lagrange multipliers, which allows only equality constraints. Similar to the Lagrange approach, the constrained maximization (minimization) problem is rewritten as a Lagrange function whose optimal point is a saddle point, i.e. a global maximum (minimum) over the domain of the choice variables and a global minimum (maximum) over the multipliers, which is why the Karush–Kuhn–Tucker theorem is sometimes referred to as the saddle-point theorem.The KKT conditions were originally named after Harold W. Kuhn and Albert W. Tucker, who first published the conditions in 1951. Later scholars discovered that the necessary conditions for this problem had been stated by William Karush in his master's thesis in 1939.

View More On Wikipedia.org
  1. T

    I Equal or larger/smaller versus larger/smaller in boundary conditions

    Hi everyone! This is the first time I'm posting on any forum and I'm still rather unsure of how to format so I'm sorry if it seems wonky. I'll try my best to keep the important stuff consistent! I am working on infinite square well problems, and in the example problem: V(x) = 0 if: 0 ≤ x ≤ a...
  2. Joa

    A Finding initial conditions

    Hello, I am trying to solve a differential equation corresponding to a visco-elastic material model consisting of 5 units of springs and dashpots connected in parallel as can be seen in the image below. I am able to come-up with a single fifth order ODE, however I am struggling to find the...
  3. maistral

    Boundary conditions for convective heat and mass transfer + wall Temperature

    I am operating via finite differences. Say for example, I have this pipe that contains a fluid. I have the boundary condition at x = x1: k is the effective thermal conductivity of the fluid, T is the temperature of the fluid at any point x, hw is the wall heat transfer coefficient, and Tw is...
  4. caters

    What is the ratio after 1 month given certain conditions?

    So this question is a math question having to do with me calculating the rate of population growth starting from a population of 100,000. I have already gotten the first 3 steps done (sex ratio, ratio of cycle time, and pregnancy ratio) after a week among those in the fertile timeframe...
  5. Quantum Alchemy

    B How constrained are the initial conditions of the universe?

    Do we know? Do we have any idea? There seems to be 2 schools of thought. 1. The initial conditions can be almost any value with universes inflating with different laws of physics. This is the multiverse and string theory 10^500 false vacua view. 2. The initial conditions are more restrained...
  6. S

    I The Multiverse and 'No boundary' conditions approach in cosmology

    Summary: Questions about the Multiverse hypothesis and the 'No boundary' conditions approach in cosmology I have some questions about James Hartle and Stephen Hawking's 'No-boundary' proposal: - In their approach multiple histories would exist. These histories could yield universes with...
  7. H

    A Crank-Nicholson method and Robin boundary conditions

    I have the following PDE I wish to solve: \frac{\partial u}{\partial t}=D\frac{\partial^{2}u}{\partial x^{2}} With the following boundary conditions: \frac{\partial u}{\partial x}(t,1)+u(t,1)=f(t),\quad u(t,0)=0 Now, I wish to do this via the Crank-Nicholson method and I would naively...
  8. Like Tony Stark

    Initial conditions in simple harmonic motion

    I have the formula for amplitude ##A=\sqrt (x_0^2 + \frac{\dot x_0 ^2}{\omega^2})##. But ##x_0## and ##\dot x_0## refers to the initial conditions, and the information that I'm given is not related to the initial conditions, or at least I'm not told so.
  9. Pencilvester

    I Violations of Energy Conditions for Metric in Relativist's Toolkit

    Here’s the metric: $$ds^2 = -dt^2+dl^2+r^2(l)d\Omega^2$$where ##r(l)## is minimum at ##l=0## with ##r(0)=r_0## and ##r## approaching ##|l|## asymptotically as ##l## approaches ##\pm \infty## Part a of the problem seemed pretty straightforward and intuitive, but part b asks which energy...
  10. Terrycho

    Partial Differential Equation: a question about boundary conditions

    Consider the following linear first-order PDE, Find the solution φ(x,y) by choosing a suitable boundary condition for the case f(x,y)=y and g(x,y)=x. --------------------------------------------------------------------------- The equation above is the PDE I have to solve and I denoted the...
  11. karush

    MHB 71 Under what conditions does the ratio A}/B equal A_x//B_x

    71.15 Two vectors $\vec{A}$ and $\vec{B}$ lie in xy plane. Under what conditions does the ratio $\vec{A}/\vec{B}$ equal $\vec{A_x}/\vec{B_x}$? Sorry but I had a hard time envisioning what this would be? also thot I posted this earlier but I can't find it
  12. maistral

    A Deriving the derivative boundary conditions from natural formulation

    PS: This is not an assignment, this is more of a brain exercise. I intend to apply a general derivative boundary condition f(x,y). While I know that the boxed formulation is correct, I have no idea how to acquire the same formulation if I come from the general natural boundary condition...
  13. Buzz Bloom

    Find the Metastatic Conditions for Omegas in the Friedmann Equation

    Required Equations Note: Includes inequalities. [1] H(s) = 0, and [2] H’(s) = 0? [3] [4] H = a’/a [5] a(t0) = a0 =1 [6] H0 = H(t0) [7] ΩR = 0 [8] Ωk<0, Ωk = -K, K > 0 [9] ΩM>1, ΩM = M, M > 1 [10] ΩΛ = 1-Ωk- ΩM = 1+K-M > 0 → M-1 < K Solution Note: I have assigned this problem to myself...
  14. N

    Is Snell's law incompatible with interface conditions? What went wrong?

    Hi everyone, Someone posted this hypothetical problem on a facebook group and I am wondering what your thoughts are. The issue is that Snell's law does not seem to hold when applied to the transmitted wave (calculated using the electromagnetic interface conditions.) Here is an example...
  15. U

    I Boundary conditions and discontinuity of EM fields

    Premise: everything that follows is done in the frequency domain. Boundary conditions If there are superficial currents (electric and magnetic) impressed on the boundary between two media, we have these discontinuities for the tangential components of the fields...
  16. V

    MHB What type of growth conditions can establish \$||f(x)||\geq K \text{dist}(x,\mathcal{X})\$?

    Hello! I have the following question: Let $f:\mathbb{R}^n\rightarrow \mathbb{R}^n$. Is there any class of class of functions and some kind of "growth conditions" such that bounds like below can be established: \begin{equation} ||f(x)||\geq g\left( \text{dist}(x,\mathcal{X})\right)...
  17. sams

    A Question about Euler’s Equations when Auxiliary Conditions are Imposed

    In the Classical Dynamics of Particles and Systems book, 5th Edition, by Stephen T. Thornton and Jerry B. Marion, page 220, the author derived Equation (6.67) from Equation (6.66) which is the following: Equation (6.67): $$\left(\frac{\partial f}{\partial y} − \ \frac{d}{dx}\frac{\partial...
  18. B

    I Finding CDF given boundary conditions (simple stats and calc)

    I'm not quite sure if my problem is considered a calculus problem or a statistics problem, but I believe it to be a statistics related problem. Below is a screenshot of what I'm dealing with. For a) I expressed f(t) in terms of parameters p and u, and I got: $$f(t)=\frac{-u \cdot a + u \cdot...
  19. A

    A Are there conditions for the vanishing of geometrical phases in QM?

    Are there theorems for sufficient and necessary conditions for the vanishing of Berry and/or Wilzeck-Zee phases for a given quantum mechanical system?
  20. maistral

    I Verification regarding Neumann conditions at time derivative

    Hi, just a question regarding neumann conditions, I seem to have forgotten these things already. I think this question is answerable by a yes or a no. So given the 2D heat equation, If I assign a neumann condition at say, x = 0; Does it still follow that at the derivative of t, the...
  21. Kaguro

    Conditions for diagonalizable matrix

    If a 3×3 matrix A produces 3 linearly independent eigenvectors then we can write them columnwise in a matrix P(non singular). Then the matrix D = P_inv*A*P is diagonal. Now for this I need to show that different eigenvalues of a matrix produce linearly independent eigenvectors. A*x = c1x A*y...
  22. N

    I Would these (unrealistic) conditions imply gravitons existed?

    So quantum states collapse when observed, ie they are interacted with, if one had an uncharged massive particle, in a true vacuum (yes, this assumption is egregious, not your usual "ignore air resistance"), and the state of the aforementioned particle collapsed, would that imply gravitons did...
  23. Othman0111

    How to deal with a circular light boundary conditions

    Hi everyone, in the attached file I tried to find the transmitted and the reflected coefficients. I ran into trouble applying the boundary conditions to the linear components of the electric field. Check the outlined boxes and see if they make sense. Thanks
  24. C

    Engineering Advanced Circuits, Laplace Transform, Find Initial Conditions

    Vo(S) = [ N(s)Vi(s) + (- s2 + s - 2) ] / s3 + s2 + 1 ; can ignore (-s^2 + s - 2). From relevant equations: Vo(S) = [N(s)*Vi(s)]/(s^3 + s^2 + 1); -> (d3Vo(t)/dt3) + (d2Vo(t)/dt2) + Vo(t) = N(t)(dvi)/dt L[vi(t)] = t to s domain: [s3Vo(s) - s2Vo(0-) - SV'o(0-) - Vo''(0-)]Vo(s) + s2 - SVo -...
  25. L

    What are the boundary conditions for this plate/ring system?

    Homework Statement [/B] I have a metal disc adhesively bonded at its edges to a piezoelectric ring. The piezoelectric ring vibrates radially which leads to the plate vibrating transversely. I am looking to work out the resonant frequency of the metal disc which I believe will depend on the...
  26. V

    MHB Limit of $(x_{n})_{n\geq 1} with Given Conditions

    Hi! I have the following sequence $$(x_{n})_{n\geq 1}, \ x_{n}=ac+(a+ab)c^{2}+...+(a+ab+...+ab^{n})c^{n+1}$$ Also I know that $a,b,c\in \mathbb{R}$ and $|c|<1,\ b\neq 1, \ |bc|<1$ I need to find the limit of $x_{n}$. My attempt is in the picture.The result should be $\frac{ac}{(1-bc)(1-c)}$ I...
  27. Cathr

    Finding Fresnel coefficients from the interface conditions

    Homework Statement We have an incident electric field, and there are two cases: 1) the field is polasised perpendicularly to the incidence plane (TE) 2) polarised in the plane (TM) Here I must be able to correctly apply the limit conditions, to find the Fresnel formulas that give the...
  28. Cathr

    I Finding the Fresnel coefficients from interface conditions

    I'm studying for an exam (so this is existential) and I never really got how to write the limit conditions for an interface. In my problem, there is an incident electric field, and there are two cases: 1) the field is polasised perpendicularly to the incidence plane (TE) 2) polarised in the...
  29. B

    Green's Function Boundary Conditions

    Homework Statement I am trying to fill in the gaps of a calculation (computing the deflection potential ##\psi##) in this paper: http://adsabs.harvard.edu/abs/1994A%26A...284..285K We have the Poisson equation: ##\frac{1}{x}\frac{\partial}{\partial x} \left( x \frac{\partial \psi}{\partial...
  30. L

    MHB Matrices- conditions for unique and no solution

    Hi,how do I go about answering the attached question? I know that for a matrix to have no solution, there needs to be a contradiction in some row. Unique solutions is when m* ${x}_{3}$ =c , where m* ${x}_{3}$ $\ne$ 0. One way I tried was if a=0, then from row (1) : b* ${x}_{3}$ =2...
  31. G

    Poisson's equation boundary conditions (electrostatics)

    Hi everyone! I have to solve a problem using Poisson's equation. There are two parallel infinite conductor planes in vacuum. The distance between them is d and they are both kept at a potential V=0. Between them there is a uniform volume density charge \rho_0>0 infinite along the directions...
  32. C

    Geiger (or other ionizing ray) Probe for Vacuum Conditions

    I am planning a physical experiement under vacuum conditions. In this experiement, I want to detect ionizing rays, especially as broad band as possible, for instance including alpha radiation. Typically geiger tubes are filled with a certain gas, that gets ionized by the radiation to measure...
  33. cromata

    Rolling with slipping and conditions for not slipping

    Suppose that we have a some rotating object (lets say a wheel with radius R). Let's observe this problem from some reference frame in which center of mass translates with some velocity v and rotates with angular velocity ω. I know that condition for rolling without slipping is v=ωR (point at...
  34. R

    Under what conditions can glass crack spontaneously?

    Are there any conditions under which glass cracks spontaneously? Just now, my mother's rear windshield cracked spontaneously. The story according to her: Slowly approaching a stop sign. No busy traffic. No trees/animals in sight. Few people. Suddenly, the rear windshield cracked with a loud boom...
  35. Krushnaraj Pandya

    Equation of a circle from given conditions

    Homework Statement Equation of the circle passing through the point (1,2) and (3,4) and touching the line 3x+y-3=0 is? Homework Equations x^2+y^2+2gx+2fy+c=0...(1) (-g,-f)=center of circle sqrt(g^2+f^2-c)=radius...(2) The Attempt at a Solution Putting (1,2) and (3,4) in equation 1 we get...
  36. B

    I Confused about the boundary conditions on a conductor

    In the textbook (attached image) it says that the boundary condition is V=0 at r=R. This creates a correlation that ##B_l=-A_l R^{2l+1}## but the potential at any boundary is continuous so when we take this account, we get. ##B_l=A_l R^{2l+1}## These two clearly contradict each other. I'd...
  37. Krushnaraj Pandya

    Conditions for conservation of momentum

    Homework Statement Consider a classic wedge and block system, (block on top of wedge(inclination theta)). there is friction between the block and wedge (not enough to prevent block from sliding). All other surfaces are smooth. For the motion that follows after releasing the block from rest, is...
  38. J

    Conditions for the applicability of u-substitution

    What are the conditions for applicability of u-substitution, i.e. when does it not work? Note that I'm not asking when is it a bad idea (that won't get you any closer to evaluating the integral), but are there any conditions that cause u-sub to yield wrong answers? I started running into what I...
  39. Mr Davis 97

    Show that given conditions, element is in center of group G

    Homework Statement Let ##G## be a finite group and ##m## a positive integer which is relatively prime to ##|G|##. If ##b\in G## and ##a^mb=ba^m## for all ##a\in G##, show that ##b## is in the center of ##G##. Homework EquationsThe Attempt at a Solution Let ##|G| = n## and ##b\in G##. Note that...
  40. Mr Davis 97

    Find which initial conditions lead to convergence

    Homework Statement Let ##b_1\in \mathbb{R}## be given and ##n=1,2,\dots## let $$b_{n+1} := \frac{1+b_n^2}{2}.$$ Define the set $$B := \{b_1\in\mathbb{R} \mid \lim_{n\to\infty}b_n \text{ converges}\}$$ Identify the set ##B##. Homework EquationsThe Attempt at a Solution I claim that ##B =...
  41. M

    MHB Romeo and Juliet initial conditions

    Hi all! I need to give a presentation on a problem in class and @countryboy helped me to figure out most of it, I only have one remaining question. Here it goes: Consider the system dr/dt = -j, dj/dt = r , where r (t) represents Romeo’s love (positive values) or hate (negative values) for...
  42. mertcan

    I Karush Khan Tucker Gradient Conditions

    Hi, initially I cut the photo off the book called Nonlinear-and-Mixed-Integer-Optimization-Fundamentals-and-Applications-Topics-in-Chemical-Engineering and page 120. My question is: in the GREEN box it says we have to use KKT gradient conditions with respect to a_i and as a result of that we...
  43. N

    Need help understanding "Electrostatic Boundary Conditions"

    We are using griffith's 4 edition in my electromagnetic course atm. and there's something I just don't understand about boundary conditions. It says that if we have a surface charge, and we put a pillbox on it, in such a way that half of it extends under the surface charge, and the other half...
  44. B

    Simple Harmonic Oscillator with Boundary Conditions

    How would you solve for the Amplitude(A) and Phase Constant(ø) of a spring undergoing simple harmonic motion given the following boundary conditions: (x1,t1)=(0.01, 0) (x2,t2)=(0.04, 5) f=13Hz x values are given in relation to the equilibrium point. Equation of Motion for a spring undergoing...
  45. MathematicalPhysicist

    A When can the Ward Identity be used in quantum field theory?

    I don't understand from Peskin when can I use Ward Identity? I mean I can see that this identity isn't always valid to use, but when it is? Take for example equation (16.10) page 508 of Peskin's and Schroeder's.
  46. C

    Equivalence of tipping conditions on an inclined plane

    We have a cube on an inclined plane. The tipping condition is the presence of an unbalanced torque relative to the center of mass (contributing forces are: the normal force and the force of friction). However, is this conditions equivalent to the previous one: The line of action of the force of...
  47. Telemachus

    A Forcing conditions in a solution of an under-determined system

    Hi there. I am solving an under-determined system of equations. The solver I've written seems to work fine, but I would like to have some control on the solution I obtain. From the infinite set of solutions of my system, I would like to use only those that fulfill some conditions. One of those...
  48. D

    I Troubles understanding slow-roll conditions in Inflation

    I've been reading up on inflation, and have arrived at the so-called slow roll conditions $$\epsilon =-\frac{\dot{H}}{H^{2}}\ll 1\; ,\qquad\eta =-\frac{\ddot{\phi}}{H\dot{\phi}}\ll 1$$ I have to admit, I'm having trouble understanding a couple of points. First, how does ##\epsilon\ll 1##...
  49. Mr Davis 97

    Conditions on H and K if H ∪ K is a subgroup

    Homework Statement Let H and K be subgroups of G. Prove that if ##H \cup K## is a subgroup of ##G## then ##H \subseteq K## or ##K \subseteq H## Homework EquationsThe Attempt at a Solution Suppose that ##H \cup K \le G##. For contradiction, suppose that neither H nor K is a subset of the...
Back
Top